Matemáticas, pregunta formulada por imperialdramonow6nm8, hace 11 meses

NECESITO AYUDA CON DERIVADAS DOY TODOS MIS PUNTOS

Adjuntos:

Respuestas a la pregunta

Contestado por IbrahimV
0

Respuesta: m=0 y n=-3

Explicación paso a paso:

Para resolver este ejercicio, vamos a plantear dos identidades que deben cumplirse:

  • si f es derivable en x=1 entonces  \lim_{n \to 1^{+}}\frac{f(x)-f(1)}{x-1}= \lim_{n \to 1^{-}}\frac{f(x)-f(1)}{x-1}
  • si f es derivable en x=1, entonces f es continua en x=1

Con respecto al primer punto, tenemos:

\lim_{x \to 1^{+}}\frac{x^{2}+3x+m-(1+3+m)}{x-1}

Luego \lim_{x \to 1^{+}}\frac{x^{2}+3x-4}{x-1}

\lim_{x \to 1^{+}}\frac{(x-1)(x+4)}{x-1}

\lim_{x \to 1^{+}}(x+4)=5

Por tanto

  • \lim_{n \to 1^{-}}\frac{f(x)-f(1)}{x-1}=5 de dónde n=-3

\lim_{x \to 1^{+}} \frac{x^{2}-nx+(n-1)}{x-1}=5

\lim_{x \to 1^{+}} \frac{(x-1)[x+(1-n)]}{x-1}=5

\lim_{x \to 1^{+}}x+(1-n)=5⇒n=-3

En relación al segundo punto, usaremos la continuidad, para poder hallar la m, sustituimos el valor de n hallado e igualamos los límites de la función por derecha e izquierda, de donde m=0

Espero te sea de utilidad, saludos.

Otras preguntas